Difference between revisions of "2022 AMC 12B Problems/Problem 15"

(Created page with "Problem: One of the following numbers is not divisible by any prime number less than 10. Which is it? <math>\text{(A)} 2^{606}-1 \text{(B)} 2^{606}+1 \text{(C)} 2^{607}-1 \tex...")
 
(Redirected page to 2022 AMC 10B Problems/Problem 17)
(Tag: New redirect)
 
(12 intermediate revisions by 5 users not shown)
Line 1: Line 1:
Problem: One of the following numbers is not divisible by any prime number less than 10. Which is it?
+
#REDIRECT [[2022_AMC_10B_Problems/Problem_17]]
<math>\text{(A)} 2^{606}-1 \text{(B)} 2^{606}+1 \text{(C)} 2^{607}-1 \text{(D)} 2^{607}+1 \text{(E)} 2^{607}+3^{607}</math>
 
 
 
Solution 1 (Process of Elimination)
 
 
 
We examine option E first. <math>2^{607}</math> has a units digit of <math>8</math> (Taking the units digit of the first few powers of two gives a pattern of <math>2, 4, 8, 6, 2, 4, 8, 6, 2, 4, 8, 6,\cdots</math>) and <math>3^{607}</math> has a units digit of <math>7</math> (Taking the units digit of the first few powers of three gives a pattern of <math>3, 9, 7, 1, 3, 9, 7, 1, 3, 9, 7, 1,\cdots</math>). Adding <math>7</math> and <math>8</math> together, we get <math>15</math>, which is a multiple of <math>5</math>, meaning that <math>2^{607}+3^{607}</math> is divisible by 5.
 
 
 
Next, we examine option D. We take the first few powers of <math>2</math> added with <math>1</math>:
 
<cmath>2^1+1=3</cmath>
 
<cmath>2^2+1=5</cmath>
 
<cmath>2^3+1=9</cmath>
 
<cmath>2^4+1=17</cmath>
 
<cmath>2^5+1=33</cmath>
 
<cmath>2^6+1=65</cmath>
 
<cmath>2^7+1=129</cmath>
 
 
 
We see that the odd powers of <math>2</math> added with 1 are multiples of three. If we continue this pattern, <math>2^{607}+1</math> will be divisible by <math>3</math>.
 
 
 
Next, we examine option B. We see that <math>2^{606}</math> has a units of digits of <math>4</math> (Taking the units digit of the first few powers of two gives a pattern of <math>2, 4, 8, 6, 2, 4, 8, 6, 2, 4, 8, 6,\cdots</math>). Adding <math>1</math> to <math>4</math>, we get <math>5</math>. Since <math>2^{606}+1</math> has a units digit of <math>5</math>, it is divisible by <math>5</math>.
 
 
 
Lastly, we examine <math>2^{606}+1</math>. Using the sum of cubes factorization <math>a^3+b^3=(a+b)(a^2-ab+b^2)</math>, we have <math>2^{606}+1^3=(2^{202}+1)(2^{404}+2^{202}+1)</math>. Since <math>2^{202}</math> ends with a <math>4</math>, and <math>4+1=5</math>, <math>2^{606}+1</math> is a multiple of <math>5</math>, which means it is divisible by <math>5</math>.
 
 
 
Since we have eliminated every option except one, <math>\boxed{\text{(C)}2^{607}-1}</math> is not divisible by any prime less than <math>10</math>.
 

Latest revision as of 00:14, 5 January 2023